Wetten auf das Erscheinen von HHT oder HTH in einer Reihe von Münzwürfen

Angenommen, wir setzen auf die Ergebnisse von Münzwürfen auf folgende Weise. Jeder von uns wählt eine andere Serie von drei Würfen, und wir werfen eine Münze, bis drei aufeinanderfolgende Würfe Ihrer oder meiner Wahl entsprechen. Zum Beispiel könnten Sie HHT wählen, während ich HTH wähle, und wir drehen bis entweder HHT oder HTH.

Wird einer von uns eher gewinnen?

Mein Freund Shimon argumentiert wie folgt. Wenn Sie HHT wählen und ich HTH wähle, wird ein langer Strom von Flips früher oder später HH enthalten, die ersten beiden Flips in Ihrer Wahl; und in diesem Moment haben Sie gewonnen, denn es gibt keine Möglichkeit, dass spätere Überschläge meinen HTH-Streak erzeugen können, bevor sie Ihren HHT-Streak erzeugen. Aber für die andere Seite existiert keine entsprechende Situation; sollte HT, die ersten beiden Flips meiner Wahl, auftreten, ist es für Sie immer noch möglich zu gewinnen, da die nächsten Flips THHT sein könnten, so dass die sechs Flips HTTHHT ein weiterer Gewinn für Sie wären.

Ich halte das für totalen Unsinn. Es ist offensichtlich, dass die nächsten drei Saltos genauso wahrscheinlich HHT oder HTT oder irgendeiner der sechs anderen möglichen Sätze von drei Flips sein werden. Sein Ansatz, zu fragen, ob Gewinne möglich sind, wenn bestimmte Folgen vergangener Flips gegeben sind, ist verblüffend, da er ohne klaren Sample-Raum arbeitet und sich der Mathematik widersetzt.

Ich schlug eine Simulation vor. Die Simulation, die ich erstellt habe, ist eine Excel-Tabelle, die Sätze von 65 Flips umdreht. In jedem Satz findet es den ersten Wurf, der mit Ihrer oder meiner Wahl beginnt, und weist einen Gewinner zu. Der folgende Screenshot zeigt diese Simulation.

Geben Sie hier die Bildbeschreibung ein

Blau wird mit HHT und Rosa mit HTH assoziiert. Im ersten Satz begann HHT mit den Flips 2, 10, 16, 20, 29, 36 und 50. HTH begann mit den Flips 3, 5, 11, 13, 21 und 51. Da HHT zuerst entstand, gibt es eine 1 im Blau senkrechter Balken neben dem ersten Satz. Wie Sie vielleicht sehen können, gewann HHT 8 Sätze der abgebildeten 10 Sätze.

In meinem kleinen Experiment gewann HHT viel öfter und oft mit dramatischen Gewinnspannen.

Ich habe mich in diesem Stapelaustausch umgesehen und ein paar Fragen gefunden, die fast genau wie meine sind – wie diese und diese – und mehrere Fragen, die meinen zumindest sehr ähnlich sind – wie diese und diese .

Sie scheinen alle zu glauben, dass Shimon Recht hat, und einige Sequenzen sind wahrscheinlicher als andere. Aber trotz dieser scheinbaren (wenn auch nicht sehr deutlich ausgedrückten) Einmütigkeit und trotz meiner eigenen Simulation kann ich es einfach nicht glauben. Offensichtlich ist jeder mögliche Satz von drei Flips gleich wahrscheinlich.

Ich denke also, meine Frage ist, kann jemand dies auf einer intuitiveren Ebene erklären und den Fehler erreichen, der mich irreführt, oder (glaube ich immer noch) Shimon?

Dies nennt sich Penney's Game und es gibt online viele Informationen darüber.
Als offensichtlicher Fall, der der Intuition helfen könnte, nehmen wir an A wählt T T T Und B wählt H T T . Dann überzeugen Sie sich selbst, dass das nur so geht A kann möglicherweise gewinnen, wenn die Münze kommt T T T zunächst also A ' S Gewinnchance ist nur 1 8 .
Beginnen Sie mit HH, der nächste Flip H ist schlecht, aber dann gefolgt von einem T gibt HHT einen Gewinn. Beginnen Sie jedoch mit HT, gefolgt von einem T (schlecht), aber keiner kann beim nächsten Flip gewinnen.
Zweifellos wurde dies auf dieser Seite schon einmal gefragt und beantwortet, wahrscheinlich mehrmals. ZB math.stackexchange.com/questions/3853515/… und math.stackexchange.com/questions/954395/… und math.stackexchange.com/questions/3856649/… und math.stackexchange.com/questions/3390215/…
@lulu Wenn also 8 Leute jede der 8 möglichen Saiten nehmen und wir alle zusammen spielen, wer gewinnt die meisten Spiele?
Das ist eine andere Frage. Jede 3 Zeichenkette wird mit gleicher Wahrscheinlichkeit angezeigt, sodass das Spiel fair (aber nicht besonders interessant) ist. Studieren Sie das Beispiel, das ich Ihnen gegeben habe, oder schlagen Sie in einer der Online-Referenzen nach.
@lulu Ich weiß nicht, wie ich Ihre Aussagen in Einklang bringen soll, dass (1) TTT HTT in nur 1 von 8 Spielen schlägt und (2) jede Zeichenfolge mit 3 Zeichen mit gleicher Wahrscheinlichkeit auftaucht. Ich hoffe, dass sich hier jemand aus den sicheren Gewässern des Jargons und der Notation herauswagt, um das kontraintuitive Ergebnis intuitiver zu erklären.
@Chaim Die einzige Möglichkeit für TTT, HTT zu schlagen, besteht buchstäblich darin, dass die ersten drei Flips TTT sind. Wenn die ersten drei Flips etwas anderes sind, gewinnt HTT. Es gibt nichts zu versöhnen ... der Gewinner wird nicht dadurch bestimmt, wessen Dreierfolge im Vakuum wahrscheinlicher ist.
Sie haben völlig Recht: Das ist völliger Unsinn. Wenn Sie und Shimon nichts anderes zu tun haben, sehen Sie sich den Anfang von "Rosencrantz and Guildenstern are dead" an.
" ... arbeitet ohne einen klaren Abtastraum und trotzt der Mathematik. " Der Abtastraum ist jedoch klar ... es ist der Raum aller Folgen von Münzwürfen, die auf HHT oder HTH enden. NB-Elemente innerhalb dieses Abtastraums sind nicht gleich wahrscheinlich, sondern haben stattdessen eine Wahrscheinlichkeit, die ihrer Länge entspricht.
@Chaim Ein ähnliches Problem wird auf Puzzling über die Intuition diskutiert und beantwortet. In diesem Puzzle, wenn es 12 gibt! Spieler, die jeweils eine andere Permutation wählen, dann ist jeder von ihnen gleich wahrscheinlich zu gewinnen (genau wie wenn Sie 8 Spieler haben, hat jeder von ihnen dieselbe Wahrscheinlichkeit zu gewinnen). Aber wenn Sie nur 2 Spieler haben, kann einer einen Vorteil haben.
@markvs Ich habe diese Show ein paar Mal gesehen, ob live oder auf Memorex. Meine Meinung zum Münzwurf war, dass, wenn R&C nicht mit der Handlung von Hamlet interagieren, die Zeit für sie nicht vergeht; sie sind einfach Geräte in der Geschichte von Hamlet. Das ununterbrochene erneute Umwerfen derselben Seite der Münze durch diese sehr große Anzahl von Versuchen symbolisierte die Idee, dass es im Wesentlichen nur ein Münzwurf ist, und sie verbringen ihre Ewigkeit im Moment dieses Umwurfs in der Schwebe.

Antworten (4)

Nehmen wir an, ich wette auf HHT und Sie wetten auf HTH. Ich behaupte, die Chancen stehen gut 2 Zu 1 zu meinen Gunsten.

Betrachten Sie es so. Früher oder später wird die Münze zum ersten Mal auf H fallen. Betrachten wir, was bei den nächsten beiden Überschlägen nach diesem ersten Kopf passiert. Es gibt vier Fälle.

  1. HHT Ich gewinne.
  2. HHH ich gewinne; früher oder später kommt die Münze T.
  3. HTH Du gewinnst.
  4. HTT Wir fangen von vorne an.

Das folgende Gleichnis kann den Fehler in Ihrem Denken erhellen oder auch nicht.

Eine U-Bahn folgt einer kreisförmigen Strecke und hält abwechselnd an den Stationen A, B, C, . . . X, Y, Z, A, B, C, . . . Sie und Ihr Freund taumeln an Bord und schlafen ein. Einige Zeit später wachen Sie beide zwischen den Stationen auf und haben keine Ahnung, wo Sie eingestiegen sind oder wo Sie sich jetzt befinden.

Sie: Ich frage mich, wo wir sind. Ich steige bei Q aus.

Ihr Freund: Ich wette mit Ihnen um zehn Dollar, dass wir meinen Stopp vor Ihrem erreichen.

Sie: Hm. Eine Station ist genauso wahrscheinlich wie die andere. Das ist eine faire Wette. Sie sind auf. Wo ist deine Haltestelle?

Dein Freund: P.

In der Tat ein schönes Beispiel. Es ist dieser Frage ähnlich .

Ich fühle Ihren Schmerz: Sie erhalten viele Argumente, die erklären, warum Ihr Freund Recht hat, und Links zu früheren Stack-Tausch-Beiträgen, die sich ebenfalls auf die Seite Ihres Freundes stellen. Sogar Ihre eigenen Simulationen bestätigen, dass Ihr Freund Recht hat! Was (wenn überhaupt!) ist jedoch falsch an Ihrer gegenteiligen Argumentation?!? Die bereitgestellten Argumente und Feststellungen können zeigen, dass Ihre Argumentation falsch ist, aber keines davon erklärt, warum oder wo Ihre Argumentation falsch ist. Anstatt also noch ein weiteres Argument zu liefern, warum die Argumentation Ihres Freundes richtig ist, lassen Sie mich sehen, ob ich auf den Punkt bringen kann, warum Ihre Argumentation falsch ist.

Ich denke, dass der Kern Ihrer Argumentation die Idee ist, dass „an jedem Punkt in einer langen Folge von Kopf und Zahl jede spezifische Folge von drei Köpfen und Zahlen genauso möglich ist wie jede andere“.

Und ja, das stimmt in dem Sinne, dass wenn Sie an diesem Punkt X in der Sequenz beginnen und nur die nächsten drei Einträge dieser Sequenz betrachten . In der Tat ist dies genau so, wie wenn Sie sagen:

Es ist offensichtlich, dass die nächsten drei Saltos genauso wahrscheinlich HHT oder HTT oder irgendeiner der sechs anderen möglichen Sätze von drei Flips sein werden.

Ob jedoch eine bestimmte Dreierfolge vor einer anderen Dreierfolge erscheint, hängt auch davon ab, welche Einträge Sie vor diesem Punkt X haben. Wir erhalten also die Argumentation Ihres Freundes: Wenn die letzten beiden Ergebnisse, die Sie vor X erhalten haben, beide Kopf waren, dann Schon der nächste Eintrag könnte das Spiel auf der Stelle beenden.

Ihr Fehler könnte also als Ergebnis einer Mehrdeutigkeit in der Behauptung angesehen werden, „ab einem bestimmten Punkt eine bestimmte Sequenz zu erhalten“. Nochmals, wenn Sie damit das Ereignis meinen, dass eine bestimmte Sequenz ab einem Punkt in der Sequenz erhalten wird, dann haben Sie Recht: Ab diesem Punkt in der Sequenz ist es genauso wahrscheinlich, dass jede Sequenz erscheint wie jede andere. Aber was wir suchen, ist das Ereignis, eine bestimmte Sequenz von diesem Punkt im Spiel zu erhalten . Und zu jedem Zeitpunkt im Spiel machen die Ergebnisse, die passiert sind, bevor Sie den Punkt der Sequenz erreicht haben, an dem Sie sich zu diesem Zeitpunkt im Spiel befinden, einen Unterschied. Ich glaube, die Verschmelzung dieser beiden unterschiedlichen Bedeutungen von „von diesem Punkt an“ liegt im Kern des Fehlers in Ihrer Argumentation.

Ihre Intuition ist insofern richtig, als dass so viele HTH wie HHT in einer ausreichend langen Folge erscheinen werden. Wenn Sie diese Vorkommnisse in Ihren Simulationen zählen, können Sie dieses Ergebnis bestätigen, und wenn Sie mit Ihrem Freund ein (langweiliges) Spiel spielen: „Wir werfen 1000 Mal eine Münze, wir zählen, wie viele HHT und wie viele HTH es gibt und wer auch immer hat die meisten Siege", ist dies ein faires Spiel.

Was Sie jedoch spielen, ist ein anderes Spiel: Sie spielen "Welche Reihenfolge kommt zuerst?"; und dieses Spiel ist nicht fair.

Sie gewinnen nicht jedes Mal, wenn HTH erscheint: Sie gewinnen, wenn HTH erscheint und HHT noch nicht erschienen ist . Ihr Gegner gewinnt, wenn HHT erscheint und HTH noch nicht erschienen ist .

Der Schlüssel hier ist zu sehen, dass die Ereignisse (die gewinnenden Fälle) nicht unabhängig sind . HHT und HTH erscheinen genauso oft, aber die Hälfte der Zeit erscheint HTH nur einen Schlag nach HHT.

Um Ihrer Intuition zu helfen, nehmen wir an, dass niemand bei den ersten drei Münzwürfen gewinnt (dies entfernt 1/4 der Versuche, die Hälfte davon gewinnt und die andere Hälfte verliert). Betrachten Sie nun das erste HTH der Sequenz und fragen Sie sich, was der vorherige Flip war:

  • Die Hälfte der Zeit wird es ein H sein, was das Muster ... HHTH macht. Sie verlieren, weil HHT nur eine Position vor HTH erschienen ist!
  • Die Hälfte der Zeit wird es ein T sein, wodurch das Muster ... THTH wird. Es hängt alles davon ab, ob HHT vorher gefunden werden soll oder nicht. Sie können gewinnen oder nicht.

Die Symmetrie ist gebrochen, weil die beiden Muster HHT und HTH nicht unabhängig sind. Die Berechnungen in den anderen Antworten sind korrekt: Die Quoten stehen 2:1 gegen Sie.

Und wenn Sie Ihren Gegner austricksen wollen, bieten Sie ein Spiel an, bei dem es darum geht, „welche Reihenfolge kommt an zweiter Stelle“ …

Ich habe bereits eine mögliche Erklärung dafür gepostet, wo und wie Ihre Intuition Sie in die Irre führt, aber vielleicht tun Sie Folgendes:

Auch hier weisen Sie zu Recht darauf hin, dass an jeder bestimmten Stelle einer langen Sequenz eine bestimmte Sequenz genauso wahrscheinlich auftritt wie jede andere. Dies impliziert auch, dass die Anzahl dieser Untersequenzen, die Sie in jeder längeren Sequenz erhalten, für jede Untersequenz genau gleich ist.

[dies ist etwas, das Sie mit Ihren Simulationen bestätigen können: Sie sollten (wenn man kleine zufällige Variationen ignoriert) genauso viele HHTs und HTHs in all den Sequenzen erhalten, die Sie sich angesehen haben.]

Daher ist es sicherlich intuitiv sinnvoll, dass sich die durchschnittliche erste Position einer Teilsequenz innerhalb einer größeren Sequenz nicht zwischen den verschiedenen Teilsequenzen unterscheidet: Es gibt genauso viele HTHs wie HHTs, also wenn wir diese einfach zufällig in die größere Sequenz streuen Warum sollte einer im Durchschnitt früher erscheinen als jeder andere?

Das scheint in die Richtung zu gehen, die Sie argumentieren, aber leider funktioniert es nicht.

Wie die Antwort von @Evargalo zeigt: Die Ereignisse von HHT und HTH, die in einer langen Folge auftreten, sind nicht unabhängig. Aber es ist auch wahr, dass das Ereignis eines HTH-Ereignisses, das in einer langen Sequenz auftritt, auch nicht unabhängig vom Auftreten eines zweiten HTH-Ereignisses ist, denn sobald Sie ein HTH-Ereignis haben, haben Sie bereits das erste H eines zweiten HTH-Ereignisses. Was tatsächlich als Ergebnis passiert, ist, dass alle HTH-Vorkommen in einer Sequenz im Vergleich zu den Vorkommen von HHT-Ereignissen tendenziell stärker geclustert/zusammengeballt sind

[Auch dies ist etwas, das Sie in Ihren Simulationen versuchen und quantifizieren könnten, aber wenn Sie nur Ihre Grafik betrachten, können Sie sehen, dass die HTH-Ereignisse stärker geclustert sind als die HHT-Ereignisse ... aber auch längere Strecken haben kein solches Vorkommen dazwischen haben ... also könnten Sie sich die Standardabweichung der Abstände ihrer Vorkommen ansehen: Sie sollten feststellen, dass sie für HTH höher ist als für HHT]

Da andererseits zwischen zwei beliebigen HHT-Sequenzen keine Überlappung möglich ist, werden sich ihre Auftritte gegenseitig "wegschieben" und gleichmäßiger verteilt sein. Und als Folge davon liegt es nahe, dass das Auftreten des ersten HTH-Ereignisses tatsächlich weiter in der Sequenz liegen würde als das erste HHT-Ereignis.